Paper 3, Section I, I

Number Theory | Part II, 2011

(i) State Lagrange's Theorem, and prove that, if pp is an odd prime,

(p1)!1modp(p-1) ! \equiv-1 \quad \bmod p

(ii) Still assuming pp is an odd prime, prove that

3252(p2)2(1)p+12modp3^{2} \cdot 5^{2} \cdots(p-2)^{2} \equiv(-1)^{\frac{p+1}{2}} \bmod p

Typos? Please submit corrections to this page on GitHub.